LSAT and Law School Admissions Forum

Get expert LSAT preparation and law school admissions advice from PowerScore Test Preparation.

 Emily Haney-Caron
PowerScore Staff
  • PowerScore Staff
  • Posts: 577
  • Joined: Jan 12, 2012
|
#45534
Hi ltowns,

The reason to reject E is because it does not provide an assumption of the argument. I think you might be approaching this without focusing on this being an assumption question, which might be what is tripping you up. Why don't you try out the assumption negation technique with E, and see if that helps clarify this for you? Let us know if you still have questions after trying that out!
 ltowns1
  • Posts: 61
  • Joined: May 16, 2017
|
#45544
Emily Haney-Caron wrote:Hi ltowns,

The reason to reject E is because it does not provide an assumption of the argument. I think you might be approaching this without focusing on this being an assumption question, which might be what is tripping you up. Why don't you try out the assumption negation technique with E, and see if that helps clarify this for you? Let us know if you still have questions after trying that out!

Hi, I' believe I'm focusing on the assumption, but I just wanted figure out if my reasoning for why the answer could be incorrect was also valid. Looking back on Daniel's explanation, I think I got my answer. Thanks!
 jennie
  • Posts: 17
  • Joined: Jul 24, 2018
|
#49027
My problem with (B) is that it didn't say all rockets will pass through both the thick lower atmosphere and the thin upper atmosphere. --> therefore, can't justify for both engines. A counterexample is exactly what the original administrator post mentioned: a rocket launched from already a high altitude.
 Adam Tyson
PowerScore Staff
  • PowerScore Staff
  • Posts: 5153
  • Joined: Apr 14, 2011
|
#49689
You're right that the author assumes that all rockets pass through both, jennie, but that means he has to assume that they all pass through the thin, upper atmosphere, right? That's not the ONLY assumption, but it is ONE assumption, and that's all we need! The negation of answer B will destroy this argument. We could also have accepted an answer that said "At some point during their ascents, all rockets will pass through the high-pressure low atmosphere," because that, too, is an assumption of the argument.

Perfect answers are not the goal on this test, and we must not allow the search for perfection prevent us from picking an answer that is good enough. Our prephrase of "all rockets do both" should be enough to allow us to pick "all rockets do one of them," because "one of them" is included in "both."
 lsatretaker
  • Posts: 17
  • Joined: Apr 13, 2019
|
#65050
Hello,

Sorry to add to the pile of discussion about E, but I'm still a bit confused, specifically with the negation, which hasn't been discussed yet. I correctly identified B as the right answer, but when I saw E immediately became doubtful of B. Bubbled B in, came back after I finished, and changed it to E.

My decision was based entirely on my negations of E and B, which when reconsidered seemed to make it obvious that E was right. Here are my negations and reasoning:

B: Not all rockets will pass through both atmospheres. (Ok, who cares; a rocket that only passes through the lower atmosphere could still work most effectively with the extra nozzle. If a rocket had both nozzles, considering that the presence of both is necessary for max effectiveness, that rocket would be able to be max effective.)

E: A rocket could work most effectively without having at least one engine with both nozzles. (Conclusion defeated; no longer are both nozzles necessary for max effectiveness.)


Thanks!
 James Finch
PowerScore Staff
  • PowerScore Staff
  • Posts: 943
  • Joined: Sep 06, 2017
|
#65399
Hi Retaker,

The big issue with (E) is that it's more or less a mere restatement of the conclusion, whereas a correct assumption will be an unstated premise that is distinct from both the given premises and conclusion. The Assumption Negation technique combines the negation of the answer choice with the given premises (as stated) in order to prove the negation of the conclusion. When using the Assumption Negation technique, make sure to negate not just the answer choice but also the conclusion, which should help clarify why (B) is correct here and (E) is not.

(B) Negated: Not all rockets pass through the upper atmosphere :arrow: Not all rockets need both long and short nozzles.

This works perfectly when combined with the premises about how the nozzles work, showing it to be correct. Contrast it to (E):

(E) Negated: A rocket need not have at least one engine with both long and short nozzles :arrow: Not all rockets need both long and short nozzles

The conclusion doesn't follow from the assumption, because it is merely a restatement of it (ie circular reasoning). This makes (E) incorrect.

Hope this clears things up!

Get the most out of your LSAT Prep Plus subscription.

Analyze and track your performance with our Testing and Analytics Package.